No CrossRef data available.
Article contents
Divisor function inequalities, entropy, and the chance of being below average
Published online by Cambridge University Press: 01 March 2017
Abstract
We extend a lower bound of Munshi on sums over divisors of a number n which are less than a fixed power of n from the squarefree case to the general case. In the process we prove a lower bound on the entropy of a geometric distribution with finite support, as well as a lower bound on the probability that a random variable is less than its mean given that it satisfies a natural condition related to its third cumulant.
- Type
- Research Article
- Information
- Mathematical Proceedings of the Cambridge Philosophical Society , Volume 163 , Issue 3 , November 2017 , pp. 547 - 560
- Copyright
- Copyright © Cambridge Philosophical Society 2017
References
REFERENCES
[1]
Alon, N., Huang, H. and Sudakov, B.. Nonnegative k-sums, fractional covers, and probability of small deviations. J. Combin. Theory Ser. B
102
3 (2012), 784–796.Google Scholar
[2]
FEDJA (http://mathoverflow.net/users/1131/fedja). lower-bound for Pr[X ≥ EX]. MathOverflow. URL:http://mathoverflow.net/q/188087 (version: 2014-11-26).Google Scholar
[3]
Friedlander, J. B. and Iwaniec, H.. Divisor weighted sums. Zap. Nauchn. Sem. S.-Peterburg. Otdel. Mat. Inst. Steklov. (POMI)
322. Trudy po Teorii Chisel (2005), 212–219, 255.Google Scholar
[4]
Hartke, S. G. and Stolee, D.. A linear programming approach to the Manickam–Miklós–Singhi conjecture. European J. Combin.
36 (2014), 53–70.Google Scholar
[5]
Khovanskiĭ, A. G.. Fewnomials. Trans. Math. Monogr. vol. 88 (American Mathematical Society, Providence, RI, 1991). Translated from the Russian by Smilka Zdravkovska.Google Scholar
[6]
Landreau, B.. A new proof of a theorem of van der Corput. Bull. London Math. Soc.
21
4 (1989), 366–368.Google Scholar
[7]
Munshi, R.. Inequalities for divisor functions. Ramanujan J.
25
2 (2011), 195–201.Google Scholar
[8]
Pokrovskiy, A.. A linear bound on the Manickam–Miklos–Singhi Conjecture. ArXiv e-prints (Aug. 2013).Google Scholar
[9]
Soundararajan, K.. An inequality for multiplicative functions. J. Number Theory
41
2 (1992), 225–230.Google Scholar
[10]
Widder, D. V.. The Laplace Transform. Princeton Mathematical Series, v. 6 (Princeton University Press, Princeton, N. J., 1941).Google Scholar
[11]
Wolke, D.. A new proof of a theorem of van der Corput. J. London Math. Soc.
s2-5 4 (1972), 609–612.Google Scholar